IBDP Maths analysis and approaches Topic: AHL 1.13 :Modulus–argument (polar) form HL Paper 1

[Maximum mark: 18]

Question:

In the following Argand diagram, the points Z1 , O and Z2 are the vertices of triangle Z1OZ2 described anticlockwise.

The point Z1 represents the complex number z1 = r1e , where r1 > 0. The point Z2 represents the complex number z2 = r2e , where r2 > 0.

Angles α, θ are measured anticlockwise from the positive direction of the real axis such that 0 ≤ α, θ < 2π and 0 < α – θ < π.

(a) Show that z1z2 * = r1r2e i(α – θ) where z2* is the complex conjugate of z2 .

▶️Answer/Explanation

Ans:

Note: Accept working in modulus-argument form

(b) Given that Re (z1 z2*) = 0, show that Z1OZ2 is a right-angled triangle.
In parts (c), (d) and (e), consider the case where Z1OZ2 is an equilateral triangle.

▶️Answer/Explanation

Ans:

(c) (i) Express z1 in terms of z2 .

▶️Answer/Explanation

Ans:

Note: Accept working in either modulus-argument form to obtain \(z_{1} = z_{2}\left ( cos\frac{\pi }{3} + isin\frac{\pi }{3} \right )\) or in Cartesian form to obtain \(z_{1} = z_{2}\left ( \frac{1}{2} + \frac{\sqrt{3}}{2} \right )i\)

(ii) Hence show that z12 + z22 = z1 z2 .

Let z1 and z2 be the distinct roots of the equation z2 + az + b = 0 where z ∈ C and a , b ∈ R.

▶️Answer/Explanation

Ans:

(d) Use the result from part (c)(ii) to show that a2 – 3b = 0.

Consider the equation z2 + az + 12 = 0, where z ∈ C and a ∈ R.

▶️Answer/Explanation

Ans:

(e) Given that 0 < α – θ < π, deduce that only one equilateral triangle Z1OZ2 can be formed from the point O and the roots of this equation.

▶️Answer/Explanation

Ans:

so (for 0 <  α – θ < π), only one equilateral triangle can be formed from point O and the two roots of this equation

[Maximum mark: 6]

Question:

Consider the complex numbers z1 = 1 + bi and z2 = (1 – b2) – 2bi, where b ∈ R, b ≠ 0.

(a) Find an expression for z1z2 in terms of b.

▶️Answer/Explanation

Ans: z1z2  = (1 + bi) ((1-b2) – (2b)i)

= (1-b2 – 2i2b2) + i (-2b + b – b3)

= (1 + b2) + i (-b – b3)

Note: Award A1 for 1+ b2 and A1 for − bi – b3i .

(b) Hence, given that arg (z1 z2) = \(\frac{\pi }{4}\) , find the value of b.

▶️Answer/Explanation

Ans:  arg (z1z2) = arctan \(\left ( \frac{-b-b^{3}}{1 + b^{2}} \right )= \frac{\pi }{4}\)

EITHER

arctan (-b) = \(\frac{\pi }{4}\) (since 1+b2 ≠, 0 for b ∈ R)

OR

-b – b3 = 1 + b2  (or equivalent)

THEN

b =−1

Question

Consider \(w = 2\left( {{\text{cos}}\frac{\pi }{3} + {\text{i}}\,{\text{sin}}\frac{\pi }{3}} \right)\)

a. These four points form the vertices of a quadrilateral, Q.

i. Express w2 and w3 in modulus-argument form.[3]

▶️Answer/Explanation

Ans: \({w^2} = 4\text{cis}\left( {\frac{{2\pi }}{3}} \right){\text{;}}\,\,{w^3} = 8{\text{cis}}\left( \pi \right)\) (M1)A1A1

Note: Accept Euler form.

Note: M1 can be awarded for either both correct moduli or both correct arguments.

Note: Allow multiplication of correct Cartesian form for M1, final answers must be in modulus-argument form.

[3 marks]

ii. Sketch on an Argand diagram the points represented by w0 , w1 , w2 and w3.[2]
▶️Answer/Explanation

Ans:

A1A1

[2 marks]

b. Show that the area of the quadrilateral Q is \(\frac{{21\sqrt 3 }}{2}\).[3]
▶️Answer/Explanation

Ans:

use of area = \(\frac{1}{2}ab\,\,{\text{sin}}\,C\) M1

\(\frac{1}{2} \times 1 \times 2 \times {\text{sin}}\frac{\pi }{3} + \frac{1}{2} \times 2 \times 4 \times {\text{sin}}\frac{\pi }{3} + \frac{1}{2} \times 4 \times 8 \times {\text{sin}}\frac{\pi }{3}\) A1A1

Note: Award A1 for \(C = \frac{\pi }{3}\), A1 for correct moduli.

\( = \frac{{21\sqrt 3 }}{2}\) AG

Note: Other methods of splitting the area may receive full marks.

[3 marks]

Let \(z = 2\left( {{\text{cos}}\frac{\pi }{n} + {\text{i}}\,{\text{sin}}\frac{\pi }{n}} \right),\,\,n \in {\mathbb{Z}^ + }\). The points represented on an Argand diagram by \({z^0},\,\,{z^1},\,\,{z^2},\, \ldots \,,\,\,{z^n}\) form the vertices of a polygon \({P_n}\).

c. Show that the area of the polygon \({P_n}\) can be expressed in the form \(a\left( {{b^n} – 1} \right){\text{sin}}\frac{\pi }{n}\), where \(a,\,\,b\, \in \mathbb{R}\).[6]
▶️Answer/Explanation

Ans: 

\(\frac{1}{2} \times {2^0} \times {2^1} \times {\text{sin}}\frac{\pi }{n} + \frac{1}{2} \times {2^1} \times {2^2} \times {\text{sin}}\frac{\pi }{n} + \frac{1}{2} \times {2^2} \times {2^3} \times {\text{sin}}\frac{\pi }{n} + \, \ldots \, + \frac{1}{2} \times {2^{n – 1}} \times {2^n} \times {\text{sin}}\frac{\pi }{n}\) M1A1

Note: Award M1 for powers of 2, A1 for any correct expression including both the first and last term.

\( = {\text{sin}}\frac{\pi }{n} \times \left( {{2^0} + {2^2} + {2^4} + \, \ldots \, + {2^{n – 2}}} \right)\)

identifying a geometric series with common ratio 22(= 4) (M1)A1

\( = \frac{{1 – {2^{2n}}}}{{1 – 4}} \times {\text{sin}}\frac{\pi }{n}\) M1

Note: Award M1 for use of formula for sum of geometric series.

\( = \frac{1}{3}\left( {{4^n} – 1} \right){\text{sin}}\frac{\pi }{n}\) A1

[6 marks]

Question

If \({z_1} = a + a\sqrt 3 i\) and \({z_2} = 1 – i\), where a is a real constant, express \({z_1}\) and \({z_2}\) in the form \(r\,{\text{cis}}\,\theta \), and hence find an expression for \({\left( {\frac{{{z_1}}}{{{z_2}}}} \right)^6}\) in terms of a and i.

▶️Answer/Explanation

Markscheme

\({z_1} = 2a{\text{cis}}\left( {\frac{\pi }{3}} \right){\text{, }}{z_2} = \sqrt 2 {\text{ cis}}\left( { – \frac{\pi }{4}} \right)\)     M1     A1     A1

EITHER

\({\left( {\frac{{{z_1}}}{{{z_2}}}} \right)^6} = \frac{{{2^6}{a^6}{\text{cis(0)}}}}{{{{\sqrt 2 }^6}{\text{cis}}\left( {\frac{\pi }{2}} \right)}}\left( { = 8{a^6}{\text{cis}}\left( { – \frac{\pi }{2}} \right)} \right)\)     M1     A1     A1

OR

\({\left( {\frac{{{z_1}}}{{{z_2}}}} \right)^6} = {\left( {\frac{{2a}}{{\sqrt 2 }}{\text{cis}}\left( {\frac{{7\pi }}{{12}}} \right)} \right)^6}\)     M1     A1

\( = 8{a^6}{\text{cis}}\left( { – \frac{\pi }{2}} \right)\)     A1

THEN

\( = – 8{a^6}{\text{i}}\)     A1

Note: Accept equivalent angles, in radians or degrees. 

Accept alternate answers without cis e.g. \({\text{ = }}\frac{{8{a^6}}}{{\text{i}}}\)

[7 marks]

Question

Given the complex numbers \({z_1} = 1 + 3{\text{i}}\) and \({z_2} = – 1 – {\text{i}}\).

a.Write down the exact values of \(\left| {{z_1}} \right|\) and \(\arg ({z_2})\).[2]

▶️Answer/Explanation

Ans:

\(\left| {{z_1}} \right| = \sqrt {10} ;{\text{ }}\arg ({z_2}) = – \frac{{3\pi }}{4}{\text{ }}\left( {{\text{accept }}\frac{{5\pi }}{4}} \right)\)     A1A1

[2 marks]

b.Find the minimum value of \(\left| {{z_1} + \alpha{z_2}} \right|\), where \(\alpha \in \mathbb{R}\).[5]
▶️Answer/Explanation

Ans:

\(\left| {{z_1} + \alpha{z_2}} \right| = \sqrt {{{(1 – \alpha )}^2} + {{(3 – \alpha )}^2}} \) or the squared modulus     (M1)(A1)

attempt to minimise \(2{\alpha ^2} – 8\alpha  + 10\) or their quadratic or its half or its square root     M1

obtain \(\alpha  = 2\) at minimum     (A1)

state \(\sqrt 2 \) as final answer     A1

[5 marks]

 

Question

(a) Find the polar form of the complex numbers

\(z=1+i\sqrt{3}\)             \(w=\frac{1}{\sqrt{2}}-\frac{1}{\sqrt{2}}i\)             \(u=-2\sqrt{3}+2i\)

▶️Answer/Explanation

Ans: \(z=2\left ( cos\frac{\pi }{3}+i\;sin\frac{\pi }3{} \right )\)    \(w=cos\left ( -\frac{\pi }{4}\right )+i\;sin\left ( -\frac{\pi }{4} \right )\)       \( u=4\left ( cos\frac{5\pi }{6}+i\;sin\frac{5\pi }{6}\right )\)

(b) Then find the polar form of

(i) \( zw\)                        (ii)\(\frac{z}{w}\)                        (iii) u3                             (iv)  \(\frac{z^{2}w^{4}}{u^{3}}\)

▶️Answer/Explanation

Ans: (i) \(zw=2\left ( cos\frac{\pi }{12}+i\;sin\frac{\pi }{12}\right )\)                              (ii)  \(\frac{z}{w}=2\left ( cos\frac{7\pi }{12}+i\;sin\frac{7\pi }{12} \right )\)

(iii) \(u^{3}=64\left ( cos\frac{5\pi }{2}+i\;sin\frac{5\pi }{2} \right )\)                (iv)  \(\frac{z^{2}w^{4}}{u^{3}}=\frac{1}{16}\left ( cos\left ( -\frac{5\pi }{6} \right )+i\;sin\left ( -\frac{5\pi }{6} \right ) \right )\)

Question

Consider \(w = 2\left( {{\text{cos}}\frac{\pi }{3} + {\text{i}}\,{\text{sin}}\frac{\pi }{3}} \right)\)

a. These four points form the vertices of a quadrilateral, Q.

i.Express w2 and w3 in modulus-argument form.[3]

▶️Answer/Explanation

Ans:

\({w^2} = 4\text{cis}\left( {\frac{{2\pi }}{3}} \right){\text{;}}\,\,{w^3} = 8{\text{cis}}\left( \pi \right)\) (M1)A1A1

Note: Accept Euler form.

Note: M1 can be awarded for either both correct moduli or both correct arguments.

Note: Allow multiplication of correct Cartesian form for M1, final answers must be in modulus-argument form.

[3 marks]

ii. Sketch on an Argand diagram the points represented by w0 , w1 , w2 and w3.[2]
▶️Answer/Explanation

Ans:

A1A1

[2 marks]

b. Show that the area of the quadrilateral Q is \(\frac{{21\sqrt 3 }}{2}\).[3]
▶️Answer/Explanation

Ans: use of area = \(\frac{1}{2}ab\,\,{\text{sin}}\,C\) M1

\(\frac{1}{2} \times 1 \times 2 \times {\text{sin}}\frac{\pi }{3} + \frac{1}{2} \times 2 \times 4 \times {\text{sin}}\frac{\pi }{3} + \frac{1}{2} \times 4 \times 8 \times {\text{sin}}\frac{\pi }{3}\) A1A1

Note: Award A1 for \(C = \frac{\pi }{3}\), A1 for correct moduli.

\( = \frac{{21\sqrt 3 }}{2}\) AG

Note: Other methods of splitting the area may receive full marks.

[3 marks]

Let \(z = 2\left( {{\text{cos}}\frac{\pi }{n} + {\text{i}}\,{\text{sin}}\frac{\pi }{n}} \right),\,\,n \in {\mathbb{Z}^ + }\). The points represented on an Argand diagram by \({z^0},\,\,{z^1},\,\,{z^2},\, \ldots \,,\,\,{z^n}\) form the vertices of a polygon \({P_n}\).

c. Show that the area of the polygon \({P_n}\) can be expressed in the form \(a\left( {{b^n} – 1} \right){\text{sin}}\frac{\pi }{n}\), where \(a,\,\,b\, \in \mathbb{R}\).[6]
▶️Answer/Explanation

Ans: 

\(\frac{1}{2} \times {2^0} \times {2^1} \times {\text{sin}}\frac{\pi }{n} + \frac{1}{2} \times {2^1} \times {2^2} \times {\text{sin}}\frac{\pi }{n} + \frac{1}{2} \times {2^2} \times {2^3} \times {\text{sin}}\frac{\pi }{n} + \, \ldots \, + \frac{1}{2} \times {2^{n – 1}} \times {2^n} \times {\text{sin}}\frac{\pi }{n}\) M1A1

Note: Award M1 for powers of 2, A1 for any correct expression including both the first and last term.

\( = {\text{sin}}\frac{\pi }{n} \times \left( {{2^0} + {2^2} + {2^4} + \, \ldots \, + {2^{n – 2}}} \right)\)

identifying a geometric series with common ratio 22(= 4) (M1)A1

\( = \frac{{1 – {2^{2n}}}}{{1 – 4}} \times {\text{sin}}\frac{\pi }{n}\) M1

Note: Award M1 for use of formula for sum of geometric series.

\( = \frac{1}{3}\left( {{4^n} – 1} \right){\text{sin}}\frac{\pi }{n}\) A1

[6 marks]

Question

If \({z_1} = a + a\sqrt 3 i\) and \({z_2} = 1 – i\), where a is a real constant, express \({z_1}\) and \({z_2}\) in the form \(r\,{\text{cis}}\,\theta \), and hence find an expression for \({\left( {\frac{{{z_1}}}{{{z_2}}}} \right)^6}\) in terms of a and i.

▶️Answer/Explanation

Markscheme

\({z_1} = 2a{\text{cis}}\left( {\frac{\pi }{3}} \right){\text{, }}{z_2} = \sqrt 2 {\text{ cis}}\left( { – \frac{\pi }{4}} \right)\)     M1     A1     A1

EITHER

\({\left( {\frac{{{z_1}}}{{{z_2}}}} \right)^6} = \frac{{{2^6}{a^6}{\text{cis(0)}}}}{{{{\sqrt 2 }^6}{\text{cis}}\left( {\frac{\pi }{2}} \right)}}\left( { = 8{a^6}{\text{cis}}\left( { – \frac{\pi }{2}} \right)} \right)\)     M1     A1     A1

OR

\({\left( {\frac{{{z_1}}}{{{z_2}}}} \right)^6} = {\left( {\frac{{2a}}{{\sqrt 2 }}{\text{cis}}\left( {\frac{{7\pi }}{{12}}} \right)} \right)^6}\)     M1     A1

\( = 8{a^6}{\text{cis}}\left( { – \frac{\pi }{2}} \right)\)     A1

THEN

\( = – 8{a^6}{\text{i}}\)     A1

Note: Accept equivalent angles, in radians or degrees. 

Accept alternate answers without cis e.g. \({\text{ = }}\frac{{8{a^6}}}{{\text{i}}}\)

[7 marks]

Question

Given the complex numbers \({z_1} = 1 + 3{\text{i}}\) and \({z_2} = – 1 – {\text{i}}\).

a.Write down the exact values of \(\left| {{z_1}} \right|\) and \(\arg ({z_2})\).[2]

▶️Answer/Explanation

Ans: \(\left| {{z_1}} \right| = \sqrt {10} ;{\text{ }}\arg ({z_2}) = – \frac{{3\pi }}{4}{\text{ }}\left( {{\text{accept }}\frac{{5\pi }}{4}} \right)\)     A1A1

[2 marks]

b. Find the minimum value of \(\left| {{z_1} + \alpha{z_2}} \right|\), where \(\alpha \in \mathbb{R}\).[5]

▶️Answer/Explanation

Ans: \(\left| {{z_1} + \alpha{z_2}} \right| = \sqrt {{{(1 – \alpha )}^2} + {{(3 – \alpha )}^2}} \) or the squared modulus     (M1)(A1)

attempt to minimise \(2{\alpha ^2} – 8\alpha  + 10\) or their quadratic or its half or its square root     M1

obtain \(\alpha  = 2\) at minimum     (A1)

state \(\sqrt 2 \) as final answer     A1

[5 marks]

Question

(a) Find the polar form of the complex numbers

\(z=1+i\sqrt{3}\)             \(w=\frac{1}{\sqrt{2}}-\frac{1}{\sqrt{2}}i\)             \(u=-2\sqrt{3}+2i\)

▶️Answer/Explanation

Ans: \(z=2\left ( cos\frac{\pi }{3}+i\;sin\frac{\pi }3{} \right )\)    \(w=cos\left ( -\frac{\pi }{4}\right )+i\;sin\left ( -\frac{\pi }{4} \right )\)       \( u=4\left ( cos\frac{5\pi }{6}+i\;sin\frac{5\pi }{6}\right )\)

(b) Then find the polar form of

(i) \( zw\)                        (ii)\(\frac{z}{w}\)                        (iii) u3                             (iv)  \(\frac{z^{2}w^{4}}{u^{3}}\)

▶️Answer/Explanation

Ans:  (i) \(zw=2\left ( cos\frac{\pi }{12}+i\;sin\frac{\pi }{12}\right )\)                              (ii)  \(\frac{z}{w}=2\left ( cos\frac{7\pi }{12}+i\;sin\frac{7\pi }{12} \right )\)

(iii) \(u^{3}=64\left ( cos\frac{5\pi }{2}+i\;sin\frac{5\pi }{2} \right )\)                (iv)  \(\frac{z^{2}w^{4}}{u^{3}}=\frac{1}{16}\left ( cos\left ( -\frac{5\pi }{6} \right )+i\;sin\left ( -\frac{5\pi }{6} \right ) \right )\)

Scroll to Top